the table below lists probabilities for the corresponding number of girls in three live births. what is the random variable, what are its possible values, and are its values all numerical?

Answers

Answer 1

The random variable in given problem is "x" . The possible values of random variable x are 0,1,2 and 3..

All possible values of random variable is numrical in nature.

We have a table to right lists probabilities for the corresponding number of girls in three live births.

from the table we cleary see that the random variable is x which represents the number of girls in three live births.

Random variable is a variable whose value is unknown or a variable which assigns a numerical value to each trials in Sample Space.

Also, we can see in list that the number of girls in three live birth values are 0, 1 ,2 and 3 ( 3 live birth)

So, the possible values of random variable x is 4 i.e., 0, 1, 2 and 3..

From data we get a result that all possible values of random variable and it's probabilities are numrical in nature.

To learn more about Random varible ,refer :

https://brainly.com/question/14356285

#SPJ4


Related Questions

Hannah prepares 44 pies for the thanksgiving feast at her school. each pie is cut into 8 pieces. approximately how many students can hannah feed? round to the nearest ten.

Answers

Answer:352

Step-by-step explanation:

Rotate the yellow dot to a location of two pi radians. After you rotate the angle, determine the value of cos 2pi, to the nearest hundredth.

Answers

Answer:

Step-by-step explanation:

The equation P = 6.31H + 41.5 can be used to predict the points earned P, buy a student on an organic chemistry final, based on the number of hours H, that he, or she studies. To earn 95 points on the final, how many hours does the model suggest someone should study?

Answers

The number of hours it is suggested to study is 8.5 hours using the concept of the equation.

What are equations?

Equations are logical assertions in mathematics that have two algebraic expressions on either side of an equals (=) sign. LHS = RHS (left-hand side = right-hand side) appears in all mathematical equations. The value of an unknown variable, or unknown quantity, can be determined by solving equations.

Given,

The given linear equation P = 6.31H + 41.5, is used to predict the points earned P.

Also P = 95,

Now, we will find the value of H by using LHS=RHS

95 = 6.31H+41.5

Subtract 41.5 from both sides,

53.5 = 6.31H

Now, divide both sides by 6.31,

8.5 = H

Therefore, the number of hours it is suggested to study is 8.5 hours.

To know more about linear equations, visit:

https://brainly.com/question/24267899

#SPJ1

PLEASE HELP

In November's Math Meet, the Little Monsters of Beast Academy solved $\frac{1}{5}$ of the problems. In March's Math Meet, they solved $\frac23$ of the problems. (There might be different numbers of problems asked in November and March.)

In total, the Little Monsters solved $13$ of $30$ problems over both Math Meets. How many problems were asked at March's Math Meet?

Answers

The number problems solved by the little Monsters  of Beast Academy in March's meet is 15.

How to determine the number of problems solved in March meet

The problem is a simultaneous equation of two unknowns with two equations.

The equations are formed as follows

let the number of questions solved by little Monsters  of Beast Academy in

March's meet be xNovember's meet be y

In November's Math Meet, the Little Monsters of Beast Academy solved $\frac{1}{5}$ of the problems = y/5

In March's Math Meet, they solved $\frac23$ of the problems = 2x/3

Hence:

y/5 + 2x/3 = 12

x + y = 30

solving the equation gives

x = y = 15

Learn more on simultaneous equation here:

https://brainly.com/question/15165519

#SPJ1

Which of the following represents an equation in standard form for the line that passes through (3, -1) and (-5, -3).
O A y-tx-1
OB 7x-4y = 1
OC. x - 4y = 7
D 4x-y=7

Answers

Answer:

Step-by-step explanation:

y = ¼x + (-7/4) is the standard form for the line that passes through (3, −1) and (−5, −3)

What is Slope intercept form?

y=mx=c is the slope intercept form of a line, m is the slope.

We need to find equation of line passing through (3, −1) and (−5, −3)

First we need to calculate slope 'm'.

slope m = y2- y1 / x2-x1

y2=-3, y1=-1, x2=-5, x1=3

m= -3 - (-1)/(-5)-3 = -2/-8=1/4

Equation of line, y=mx+c

find C in the equation (y=mx+c)

C = y-mx

C = -1-1/4(3)= -7/4

now replace the values you find in the equation (y=Mx+C)

y =1/4x-7/4

Hence y =1/4x-7/4 is the equation of line  passes through (3, −1) and (−5, −3).

3X + 4X - 2 = 12
Please help again I’m not very smart lol

Answers

[tex]{ \pink{ \boxed{ \blue{ \sf{x = 2}}}}}[/tex]

Step-by-step explanation:

[tex]{ \red{ \sf{3x + 4x - 2 = 12}}}[/tex]

[tex]{ \red{ \sf{7x = 12 + 2}}}[/tex]

[tex]{ \red{ \sf{7x = 14}}}[/tex]

[tex]{ \red{ \sf{x = \frac{14}{7}}}} [/tex]

[tex]{ \blue{ \boxed{ \green{ \sf{x = 2}}}}}[/tex]

you can only add the numbers together that have (x)

If g(x) = -2x² + 3x - 1, find g(x + 1)

Answers

The value of the equation g(x+1) is -2x²-x

Given equation,

g(x) = -2x²+3x-1

Replacing x in the equation by x+1

g(x+1) = -2(x+1)² +3(x+1) -1

g(x+1) = -2(x²+1+2x) +3x +3 -1

g(x+1) = -2x² -2 -4x +3x +3 -1

g(x+1) = -2x²- x

Hence the final answer is g(x+1) = -2x²-x

Since this, the question says to find the value g(x+1) we need to replace the value of x in the given equation g(x) by x+1 to find value. After replacing we get -2(x+1)² +3(x+1) -1. Next, we need to solve the bracket (x+1)² which will give us x²+1+2x. We will multiply this by -2, which will give us -2x² -2 - 4x. Next, we will solve 3(x+1) which will give us 3x+3. Finally, we need to solve the whole equation to get the answer -2x²- x

I hope this answer helps.

To learn more about composite functions,

https://brainly.com/question/20379727

the student council at a local college has 8 seniors, 7 juniors, 5 sophomores and 3 freshmen. if 3 of them are randomly chosen to be on a committee, what is the probability that none of them are seniors?

Answers

The probability is 0.6 that if randomly choosen none of them are senior.

What is probability?The area of mathematics known as probability deals with numerical representations of the likelihood that an event will occur or that a statement is true. An event's probability is a number between 0 and 1, where, roughly speaking, 0 denotes the event's impossibility and 1 denotes certainty.  The likelihood that an event will occur increases with its probability. A straightforward illustration is tossing a fair (impartial) coin. The chance of both outcomes ("heads" and "tails") is equal because the coin is fair, "heads" is more likely than "tails," there are no other conceivable outcomes, and the likelihood of either outcome is half .

acc to our question-

total people in council= 23The number of committees that have no seniors: In this case, we have to select  5  people from a group of  1+3+4=8  people. This can be done in  8C5  ways.The number of committees that have no seniors: In this case, we have to select  5  people from a group of  1+3+4=8  people. This can be done in  8C5  ways.

Hence,the probability is 0.6 that if randomly choosen none of them are senior.

learn more about probability click here:

https://brainly.com/question/13604758

#SPJ4

Which answer is correct?

(A) y = 3x − 2
y = 2x + 5
(B) y = −3x + 2
y = 2x − 5
(C) y = −3x − 2
y = −2x − 5
(D)y = 3x + 2
y = −2x + 5

Answers

Step-by-step explanation:

the slow of a line is the factor of x in a "y = ..." firm of equation.

the line going through (1, 5) has a slope of 3 (when x increases by +1, then y increases by +3, so the slope y/x = 3).

(B) and (C) are out, because there is no line with slope 3 (only -3).

the line going through (-2, 9) has a slope of -2 (when x increases by 1, y decreases by 2, so the slope y/x = -2/1 = -2).

(A) is out, as there is no line with the slope -2 (only +2).

so, (D) is the correct answer.

The tables represent two linear functions in a system. A 2 column table with 5 rows. The first column, x, has the entries, negative 4, negative 2, 0, 2. The second column, y, has the entries, 26, 18, 10, 2. A 2 column table with 5 rows. The first column, x, has the entries, negative 4, negative 2, 0, 2. The second column, y, has the entries, 14, 8, 2, negative 4. What is the solution to this system? (1, 0) (1, 6) (8, 26) (8, –22) Mark this and return

Answers

From the given table which represents the two system of linear function is given by y = -4x +10 and y = -3x +2 , the solution of the system of linear function is (x, y) = (8 , -22).

As given in the question,

Table which represents the system of linear function are as follow:

table 1:

x:      -4       -2        0       2

y:     26       18       10     2

table 2:

x:      -4       -2        0       2

y:     14       8         2      -4

For table1 :

System of linear function:

(x₁ , y₁) = (0,10)

(x₂ , y₂) = (2, 2)

Equation of linear function:

(y -y₁)/ (x-x₁) =(y₂ -y₁)/(x₂ -x₁)

⇒(y -10)/ (x-0) =(2-10)/(2-0)

⇒y-10 =-4(x)

⇒ y = -4x +10     __(1)

For table2 :

System of linear function:

(x₁ , y₁) = (0,2)

(x₂ , y₂) = (2, -4)

Equation of linear function:

(y -y₁)/ (x-x₁) =(y₂ -y₁)/(x₂ -x₁)

⇒(y -2)/ (x-0) =(-4-2)/(2-0)

⇒y-2 =-3(x)

⇒ y = -3x +2     __(2)

Solve system of linear function (1) and (2) we get,

-4x +10 = -3x +2

⇒4x -3x =10 -2

⇒x =8

⇒y = -4(8) +10

     = -22

(x, y) = (8 ,-22)

Therefore, from the given table which represents the two system of linear function is given by y = -4x +10 and y = -3x +2 , the solution of the system of linear function is (x, y) = (8 , -22).

Learn more about linear function here

brainly.com/question/21107621

#SPJ1

Answer:

D

Step-by-step explanation:

trust

Eric's city took a telephone poll about a plan to build a new hotel downtown. 200 people took
the poll. 25% of them were in favor of the new hotel. How many people were in favor of the
new hotel?

Answers

Answer:

 50 people

Step-by-step explanation:

Solution

25% as a fraction is 25/100 which reduces down to 1/4. What this tells you is that 1/4 of the people asked want the hotel

1/4 * 200 = 50

50 people want the hotel.

on average dog bite victims arrive at carver memorial hospital at a rate of 2 victims per day. which probability distribution is most nearly appropriate to describe the number of dog bite victims who arrive at this hospital on a given day?

Answers

The "Poisson distribution" is the probability distribution that comes the closest to accurately describing that many dog bite patients visit this hospital across any given day.

Describe the term Poisson distribution?A discontinuous probability distribution is a Poisson distribution. It provides the likelihood that a response will occur a specific number of times (k) over a specific duration or area. The mean number of occurrences, designated by the letter "lambda," is the single parameter of the Poisson distribution.The chance of a discrete (i.e., countable) result is provided by a Poisson distribution, that is an discrete probability distribution.The discrete result for Poisson distributions is k, that also stands for the frequency of an event.

For the given data of,

An average of two dog bite victims show up at Carver Memorial Hospital each day.

A probability distribution that most accurately predicts the number of dog bite victims who visit this hospital on any given day is characterized as the "Poisson distribution."

To know more about the Poisson distribution, here

https://brainly.com/question/9123296

#SPJ4

Find the equation of the line that is parallel to another line whose equation is x+2y+8=0 and passes though the point(-2,-3)

Answers

There is no line parallel to the line x + 2 · y + 8 = 0, as point (x, y) = (- 2, - 3) lies on the original line.

How to determine the equation of a line parallel to another line and that passes through a given point.

In this problem we need to determine the equation of line based on its slope relationship respect to another line and a given point. Two lines are parallel if and only if they have the same slope (m) and two distinct intercepts (b). First, transform the given equation of the line into slope-intercept form:

x + 2 · y + 8 = 0

2 · y = - x - 8

y = - (1 / 2) · x - 4

Then, the resulting line has a slope of - 1 / 2.

Second, determine the intercept of the resulting line:

y = m · x + b

b = y - m · x

b = - 3 - (- 1 / 2) · (- 2)

b = - 3 - 1

b = - 4

The intercept of the resulting line is - 4.

Third, transform the equation of the resulting line into standard form:

y = - (1 / 2) · x - 4

- 2 · y = x + 8

x + 2 · y + 8 = 0

There is no parallel line for the point (x, y) = (- 2, 3).

To learn more on parallel lines: https://brainly.com/question/2456036

#SPJ1

r is a relation on the set of all nonnegative integers. (a,b) is in r if a and b have the same remainder when divided by 5

Answers

The relation accepts reflexive, symmetry, and transitive.

Recall that a relation R is reflexive if the element (x, x) belongs to R for all elements X in the domain of R.

If (x, y) belongs to R, then follows that (y, x) must likewise belong to R, making the situation symmetric.

And it is transitive if (x, y) and (y, z) belongs to R necessarily implies that (x, z) belongs to R.

Given r is a relation on the set of all nonnegative integers R(a,b)

Reflexive - YES. A given number a will always have the same remainder when divided by 5.

Symmetric - YES. If a and b have the same remainder when divided by 5, then b and a are the same pair, so again they will have the same remainder.

Transitive - YES. If a and b as well as b and c have the same remainder when divided by 5, this is possible if both a and c also have the same remainder when divided by 5.

Therefore the relation accepts reflexive, symmetry, and transitive.

To learn more about reflexive & symmetry visit

https://brainly.com/question/3014815

#SPJ4

the first step of writing an equation when given two points is the same for both strategies, slope-intercept or point-slope forms. What would be the first step in finding the equation of the line that passes through (5, 1) and (3, 5)? create a graph using the two points find the slope find the y-intercept substitute one point’s coordinates into either form

Answers

The answer to this question is provided in main body.The equation will be as,

What is equation?

an equation could be a formula that uses the sign to attach 2 expressions to indicate that they're equal.

Main Body:

given : points (5,1) and (3,5).

Since slope of line passing through two points

(x₁,y₁ ) and (x ₂ ,y₂​ ) is m= (x₂​ −x₁)/(y₂​ −y₁)

​We now find the slope of the line passing through the points (5,1) and (3,5) as shown below:

⇒m= (5-1)/(3-5)

⇒m= 4/-2

⇒m= -2

Therefore, the slope of the line is  -2

Now use the slope and point (5,1) to find the y-intercept.

y= mx+b

⇒1= -2(5) +b

⇒1= -10 + b

⇒b= -11

Write the equation in slope intercept form as:

y= mx+b

y = -2x -11

Hence the equation of line is, 2x + y= -11.

To learn more about equation click on the link below

https://brainly.com/question/22688504

#SPJ9

Rewrite in simplest terms: -5(-6k-6)-(-9k-10)−5(−6k−6)−(−9k−10)
help please n ty

Answers

The value of the expression -5(-6k-6)-(-9k-10) in simplest terms is 39k + 40

What is an algebraic expression?

An algebraic expression can be described as an expression that is made up of terms, variables, coefficients, factors and constants.

These expressions are also known to be composed of certain mathematical operations, such as;

SubtractionDivisionMultiplicationBracketAdditionParentheses

We have the expression;

-5(-6k-6)-(-9k-10)

expand the bracket

-5(-6k - 6) + 9k + 10

collect like terms

30k + 30 + 9k + 10

Add the like terms

39k + 40

Hence, the value is 39k + 40

Learn more about algebraic expressions here:

https://brainly.com/question/4344214

#SPJ1

A couple buys a $290000 home, making a down payment of 15%. The couple finances the purchase with a 15
year mortgage at an annual rate of 3.4%. Find the monthly payment.
per month.
If the couple decides to increase their monthly payment to $1950, find the number of payments.
payments (give 2 decimal places)
which we would round up to
total payments.

Answers

The monthly payment for the first condition is $1750.11 and the total number of payments is 156.

What is the monthly payment?

Monthly payment is defined as the amount paid every month depending on the terms of the mortgage loan. This includes the principal, which is the actual balance on the loan, and the interest on the loan.

The formula for monthly payments on the mortgage is given:

                                [tex]MP = \frac{P(\frac{r}{n}) }{1- (1+\frac{r}{n})^{-nt} }[/tex]

where, MP = monthly payment

            P = principal amount

             r = rate of interest

             n = number of payments per year

              t = number of years

Given, a couple buys a house for $290000 and makes a down payment of 15%

Down payment = 0.15*290000 = $ 43500

First condition,

Present value of loan (P) = $290000 - $43500 = $246500

rate of interest (r) = 3.4% = 0.034

number of payments per year (n) = 12

number of years (t) = 15 years

Then, the monthly payment is calculated by:

             [tex]MP = \frac{246500(\frac{0.034}{12}) }{1- (1+\frac{0.034}{12})^{-12*15} } = 1750.11[/tex]

Therefore, monthly payment = $1750.11

Second condition,

monthly payment (MP) = $1950

Present value of loan (P) = $290000 - $43500 = $246500

rate of interest (r) = 3.4% = 0.034

number of payments per year (n) = 12

time in years (t) = x years

Then,

or,  [tex]1950 = \frac{246500(\frac{0.034}{12}) }{1- (1+\frac{0.034}{12})^{-12*x} }[/tex]

or, [tex]x = 13.06 = 13[/tex]

So, the total number of payments =  13*12 = 156

To learn more about monthly payment

https://brainly.com/question/22323087

#SPJ1

help meeeeeeeeeeeeee pleaseeeeeeeeee!!!!!!!!!!!!!!!!!!!!!!!!!!!!!!!!!!!!!!!!!!!!!!!

Answers

Answer:

0.64 feet

Step-by-step explanation:

make h the subject of this expression

[tex]t = \sqrt{ \frac{h}{16} } [/tex]

[tex] {t}^{2} = \frac{h}{16} [/tex]

[tex]h = 16 {t}^{2} [/tex]

then substitute in the value of t=0.2 into the equation of h:

h = 16 × (0.2)² = 0.64 feet(answer)

Two integers, a and b, have different signs. They absolute value of integer a is divisible by the absolute value of integer b. Find two integers that fit this description. Then decide if she product of the integers is greater than or less than the quotient of the integers. Show your work.

Answers

Answer:

Step-by-step explanation:

One example is a=-6 and b=3

-6/3=-2

-6*3=-18

-2 is greater than -18; the quotient is greater than the product in this case.

Another example is a=6 and b=3

6/3=2

6*3=18

2 is smaller than 18; the quotient is smaller than the product in this case.

What is the image of (1, 0) after a counterclockwise rotation of 60°?

Answers

Answer: I'm pretty sure this is the same question. https://brainly.com/question/16428516

A probablity experiment is conducted in which the sample space of the experiment is S= (5. 6, 7, 8,9, 10, 11, 12, 13, 14, 15. 16}, event F={8,9,10,11,12,13), And event G = {12, 13, 14, 15} Assume that each outcome is equally likely. List the outcomes in F For G. Find P(F or G) by counting the number of outcomes in F or G. Determine P(F or G) using the general addition rule.

List the outcomes in F or G.

A. For G= {__}

(Use a comma to separate answers as needed)

Answers

Using the general addition rule, p(F or G) = 0.667.

What is sample space?

In probability theory, the set of all feasible outcomes or outcomes of an experiment or random trial is referred to as the sample space. The sample points, or potential ordered outcomes, are listed as elements in the set used to represent the sample space.

Let,

S = {5, 6, 7, 8, 9, 10, 11, 12, 13, 14, 15, 16}

F = {8, 9, 10, 11, 12, 13}

G = {12, 13, 14, 15}

n(S) = number of elements in set S are 12.

n(F) = 6

n(G) = 4

n(F and G) = common elements in F and G = 2

n(F or G) = n(F) + n(G) - (F and G)

n(F or G) = 6 + 4 - 2

n(F or G) = 8

Now, using the general addition rule,

P(F or G) = P(F) + P(G) - P(F and G)

Here,

P(F) = n(F)/n(S) = 6/12 = 1/2

P(G) = n(G)/n(S) = 4/12 = 1/3

P(F and G) = n(F and G)/n(S) = 2/12 = 1/6

⇒ P(F or G) = 1/2 + 1/3 - 1/6

                   = 5/6 - 1/6

                   = 4/6

⇒ P(F or G) = 0.667

Hence, using the general addition rule, P(F or G) = 0.667.

To know more about the sample space, click on the link

https://brainly.com/question/10558496

#SPJ1

ABCD, BEG and CEF are straight lines. Find a and b

Answers

Answer:A=72

B=108

Step-by-step explanation:

The solution is, angle a = 72 & b = 108

What is an angle?

In Plane Geometry, a figure which is formed by two rays or lines that shares a common endpoint is called an angle. The two rays are called the sides of an angle, and the common endpoint is called the vertex.

here, we have,

given that,

from the given figure we get,

∠ABC = 180

∠BCD = 180

so, we have,

∠ABC + ∠BCD = 180 + 180

ABE + EBC + a + b = 360

again, we know that,

EBC + a = 180 - 38 = 142

[we know, from property of triangle's angle]

now, we get,

ABE + 142 + b = 360

or, 110 + 142 + b = 360

or, b = 108

so, we get,

a = 180 - 108 = 72

Hence, The solution is, angle a = 72 & b = 108.

To learn more on angle click:

brainly.com/question/28451077

#SPJ2

26. Use Patterns and Structure
Amoli says you should use
the Distributive Property to
multiply the two expressions.
Yuji says you should use the
Associative Property. Who is
correct? Explain
(125+y) 1/2y

Answers

Amoli is Correct, where distributive property is used to multiply two expressions.

What is Distributive property?

The Distributive Property is an algebra property which is used to multiply a single term and two or more terms inside a set of parentheses.

What is Associative property?

A mathematical principle known as the associative property states that grouping the factors in a multiplication problem does not affect the solution.

Step-by-step explanation:

According to distributive property:

= (125 * (1/2y)) +( y * (1/2y))

= 125/2y + 1/2

Here, Associative property does not held.

Hence, Distributive property should be used.

To know more about distributive and associative property check given link:

https://brainly.com/question/2807928

#SPJ1

WORTH 100 POINTS

Write [tex]\frac{3}{2}[/tex] As a Tape Diagram

Answers

Answer:

Step-by-step explanation:

Dibuja una diagrama de cinta dividido en 3 y pinta 2 de los espacios

De nada :>

from a group of 12 students, we want to select a random sample of 5 students to serve on a university committee. how many combinations of random samples of 5 students can be selected?

Answers

The number of combinations of random samples of 5 students can be 792.

Given, 12 groups of students

We have to select random samples of 5 students,

Here we will use the combination formula,

Cₙ,ₓ = n! / x! (n - x)!

Here n=12 and x=5

Cₙ,ₓ = 12! / 5! (7!)

Cₙ,ₓ = 479,001,600 / (120) × (5040)

Cₙ,ₓ = 792

Therefore the possible ways of selecting the random samples of 5 students are 792.

To learn more about permutations and combinations

https://brainly.com/question/13387529

#SPJ4

rationalise the denominator of \frac{26}{4+\sqrt{3}}

Answers

The rationalization of the denominator of the given fraction gives 8 -2√3

Rationalizing the denominator

From the question, we are to rationalize the denominator of the given fraction

The given fraction is

26 / (4+√3)

To rationalize the given expression, we will multiply both the numerator and the denominator of the fraction by the conjugate of the denominator.

The denominator is 4+√3

Thus,

The conjugate of the denominator is 4-√3

Now, multiply the numerator and the denominator by the conjugate.

That is,

(26 × (4-√3)) / ((4+√3) × (4-√3))

(104 - 26√3) / (16 - 3)

(104 - 26√3) / 13

= 104/13 - (26√3)/13

= 8 - 2√3

Hence,

The result of rationalizing is 8 -2√3

Learn more on Rationalizing the denominator here: https://brainly.com/question/25222850

#SPJ1

help meeeeeeeeeeee pleaseeeeeeeeeee rn rnnnnnnnnn!!!!!!!!!!!!!!!!!!!!!!!!!!!!!!!!!!!!!!!!!!!!!!
help meeeeeeeeeeee pleaseeeeeeeeeee rn rnnnnnnnnn!!!!!!!!!!!!!!!!!!!!!!!!!!!!!!!!!!!!!!!!!!!!!!

Answers

The Maximum and Minimum values of x is  = (3,30)

The maximum of a quadratic function occurs at

x = − b/2a

If  a is negative, the maximum value of the function is

f ( − b/2a).

f max x  =ax^2+bx+c occurs at

x = −b/2a

Finding the value of

x = −b/2a

Substitute in the values of a and b.

x = − 18/2(3)

Simplify

x = -3

Replace the variable x with -3 in the expression

f (-3) = -3(-3)^2+18(-3)+3

f(-3) = - 27 +54 +3

f(-3) = -30

f(3) = 30

The maximum   of -3x^2+18x+3 = (3,30)

Learn more about the maximum value here :

https://brainly.com/question/14316282

#SPJ1

draw the full decision tree for the parity function of four boolean attributes, a, b, c, and d. is it possible to simplify the tree? g

Answers

The diagramfull decision tree for the parity function of four boolean attributes, a, b, c, and d is shown below picture and  it is possible to simplify the tree.

learn more about decision tree here

https://brainly.in/question/13507974

#SPJ4

Solve each system by substitution.
4x + y = 19
6x + 7y=1

Answers

Step-by-step explanation:

that simply means we use one equation to express one variable by the other, and then we use that identity in the second equation (substituting one variable by the expression of the other variable. and then we solve for that remaining variable.

finally we use one of the original equations to solve for the second variable.

4x + y = 19

so, that gives us

y = 19 - 4x

we use that now in the second equation :

6x + 7(19 - 4x) = 1

6x + 133 - 28x = 1

-22x + 133 = 1

-22x = -132

22x = 132

x = 6

now, let's use e.g.

4x + y = 19

4×6 + y = 19

24 + y = 19

y = -5

The volume of the triangular prism is 12.5 m³. The length is 2.5 m and the base is 2 m. What is the width?

Answers

Answer:

  (4)  5 m

Step-by-step explanation:

You want the length of side x of a right triangular prism with base edge lengths of 2.5 m and 2 m, and a volume of 12.5 m³.

Volume

The volume of the prism is given by the formula ...

  V = Bh

where B is the area of the base:

  B = 1/2bh . . . . where b and h are the leg dimensions of the right triangle

Using these formulas together, we have ...

  V = 1/2(2.5 m)(2 m)x

  12.5 m³ = 2.5x m²

Dividing by 2.5 m², we find x to be ...

  (12.5 m³)/(2.5 m²) = x = 5 m

The dimension labeled x has length 5 meters.

Other Questions
how should you approach public communication differently from group communication? by preparing for your audience to interrupt you and direct the conversation by considering the background and dynamics of your audience before preparing your message by assuming a greater level of intimacy between you as the speaker and your audience with an awareness that the audience is more likely to defer to the speaker in a public communication context Jerald is having drain issues at his home and decides to call a plumber. The plumber charges $35 to come to his house and $60 for every hour they work. If the plumber charges Jerald a total of $305, how many hours did the plumber work?Write and solve an equation to determine the number of hours worked by the plumber. 35x + 60 = 305; x = 7 hours 35x 60 = 305; x = 10.4 hours 60x + 35 = 305; x = 4.5 hours 60x 35 = 305; x = 5.7 hours Find the equation of the vertical line that passes through point M(2, 6) the red bud company pays a constant dividend of $3.00 a share. the company announced today that it will continue to do this for another 2 years after which time they will discontinue paying dividends permanently. what is one share of this stock worth today if the required rate of return is 8.6 percent? insertion of additional repeats in repetitive segments of dna often is a result of what? translation errors replication slippage chromosome rearrangements transformation transcription slippage epidemiologic studies of the role of a suspected factor in the etiology of a disease may be observational or experimental. the essential difference between experimental and observational studies is: IF YOU PLAY BASKETBALL HELP ME PLEASE AND AWNSER THESE QUSTIONS purchasing of services can be more challenging than purchasing of materials because: group of answer choices No Name Woman by Hong KingstonWhy did Kingstons mother tell her the story about her aunt?A. To see if she could keep a secretB. To console herC. To warn herD. To congratulate her Please help please and thank you.. an older adult is postoperative day one, following a coronary artery bypass graft (cabg). the client's family members express concern to the nurse that the client is uncharacteristically confused. after reporting this change in status to the health care provider, what additional action should the nurse take? Temperature, humidity, light and atmospheric pressure are Which equation show the relationship between the number ofhours, x, and the number of dogs groomed, y?y = 15xy = 5xy = 30xy=3x When f(x) equals X -7/2, what is the value of (FF^-1) (3) A study was commissioned to find the mean weight of the residents in certain town. The study found the mean weight to be 193 pounds with a margin of error of 9 pounds. Which of the following is not a reasonable value for the true mean weight of the residents of the town? Write a system of equations to describe the situation below, solve using any method, and fillin the blanks.Sidney wants to take group fitness classes at a nearby gym, but needs to start by selecting amembership plan. With the first membership plan, Sidney can pay $36 per month, plus $2 foreach group class she attends. Alternately, she can get the second membership plan and pay$35 per month plus $3 per class. If Sidney attends a certain number of classes in a month,the two membership plans end up costing the same total amount. How many classes permonth is that? What is that total amount?If Sidney attendsSubmitclasses per month, each membership plan costs $ You are given 13.0 m of thin wire. You form the wire into a circular coil with 70 turns.If this coil is placed with its axis parallel to a 0.19 T magnetic field, what is the flux through the coil? what are some media literacy antonyms? an office space is 75 ft wide by 100 ft long and 10 ft high. the indoor conditions are 75 f and 24% rh, and the outside conditions are of and saturated (100% rh). the space is designed for 50 people. the infiltration rate estimated to be 0.75 ach. calculate the sensible heat loss due to ventilation and infiltration. what is the thermal efficiency of a rankine cycle with isentropic compression and expansion for which steam leaves the boiler as saturated vapor at 6 mpa and is condensed at 50 kpa? how does this efficiency compare the theoretical carnot efficiency? hint: draw a thermodynamic sketch of the process and look online for useful steam tables.